0
$\begingroup$

Implement the Quantum Phase Estimation (QPE) algorithm using the unitary operator S and the eigenstate∣1⟩ as the target qubit. Use 3 counting qubits and 1 target qubit.Assumptions / notation:Assume the unitary S satisfiesS∣1⟩ = exp (2πiφ) ∣1⟩for some unknown phase φ∈[0,1). Your QPE circuit must estimate φ to 3-bit precision (i.e., output the best3-bit binary approximation of φ).

I am confused with the target Qubit and when I am taking the 3 Qubit then it is having 12.5% Probability for each as an output? Please any suggestion is helpful

askedOct 30 at 11:00
Anitya Gupta's user avatar
$\endgroup$

1 Answer1

0
$\begingroup$

Target qubit is eigenstate of the unitary S.In QPE you can estimate$\theta$ by$\theta=\frac{j}{2^m}$ where$j \in \{0, 1, 2, .., 2^m-1\}$. Here$j$ is a measured value from your counting 3 qubits, so$m=3$. Hence it has 3-bit precision.

answeredOct 30 at 14:06
taketoshi kinoshita's user avatar
$\endgroup$

Your Answer

Sign up orlog in

Sign up using Google
Sign up using Email and Password

Post as a guest

Required, but never shown

By clicking “Post Your Answer”, you agree to ourterms of service and acknowledge you have read ourprivacy policy.

Start asking to get answers

Find the answer to your question by asking.

Ask question

Explore related questions

See similar questions with these tags.